In a $\triangle ABC$, if $L$ and $M$ are points on $AB$ and $AC$ respectively such that $LM \| BC$.
Prove that $ar(\triangle LOB) = ar(\triangle MOC).


Given:

In a $\triangle ABC$, $L$ and $M$ are points on $AB$ and $AC$ respectively such that $LM \| BC$.

To do:

We have to prove that $ar(\triangle LOB) = ar(\triangle MOC)$.

Solution:

Join $LM, LC$ and $MB$.


$L$ and $M$ are the mid points of $AB$ and $AC$.

This implies,

$LM \| BC$

$\triangle LBM$ and $\triangle LCM$ are on the same base $LM$ and between the same parallels.

Therefore,

$ar(\triangle LBM) = ar(\triangle LCM)$......…(i)

$ar(\triangle LCM) = ar(\triangle LBM)$

$\triangle LBC$ and $\triangle MBC$ are on the same base $BC$ and between the same parallels.

Therefore,

$ar(\triangle LBC) = ar(\triangle MBC)$......…(ii)

$ar(\triangle LMB) = ar(\triangle LMC)$              [From (i)]

$ar(\triangle ALM) + ar(\triangle LMB) = ar(\triangle ALM) + ar(\triangle LMC)$                 [Adding $ar(\triangle ALM)$ on both sides]

$ar(\triangle ABM) = ar(\triangle ACL)$

$ar(\triangle LBC) = ar(\triangle MBC)$                  [From (ii)]

$ar(\triangle LBC) - ar(\triangle BOC) = ar(\triangle MBC) - ar(\triangle BOC)$

$ar(\triangle LBO) = ar(\triangle MOC)$

Hence proved. 

Tutorialspoint
Tutorialspoint

Simply Easy Learning

Updated on: 10-Oct-2022

27 Views

Kickstart Your Career

Get certified by completing the course

Get Started
Advertisements